subject
Mathematics, 12.06.2020 13:57 houtchhaytang

Here is a list of numbers. 12 13 19 16
32 15 13
a) Work out the range of the numbers in the list.
b) Work out the median of the numbers in the list.

ansver
Answers: 1

Other questions on the subject: Mathematics

image
Mathematics, 21.06.2019 15:00, jfarley259
Hye ryung is the president of the local chapter of the american medical students association (amsa). she is organizing local outreach and informational meetings. at her first meeting there are five people present (including herself). every month after that her group grows by 5 people. a. how many members are in the group after 6 months? b. the logistics of hosting and feeding her group at meetings was more complicated than expected. the cost of feeding 5 people at her first meeting was $30, for 10 people it was $35, for 15 it was $45, and after 6 months all of the costs had added up to $100. write a function to model the cost with the number of people attending meetings. c. what connections can you make between the linear growth of the group’s membership and the increasing costs of running meetings?
Answers: 3
image
Mathematics, 21.06.2019 18:30, esta54
Angela took a general public aptitude test and scored in the 88th percentile for aptitude in accounting. what percentage of the scores were at or below her score?
Answers: 2
image
Mathematics, 21.06.2019 22:30, bradenhale2001
I’m really confused and need your assist me with this question i’ve never been taught
Answers: 1
image
Mathematics, 22.06.2019 01:30, Falconpride4079
Drag the tiles to the correct boxes to complete the pairs. match the exponential functions with their horizontal asymptotes and y-intercepts. f(x) = 7x − 4 f(x) = 3x+2 + 4 f(x) = 9x+1 − 4 f(x) = 2x + 4 horizontal asymptote and y-intercept exponential function horizontal asymptote: y = 4 y-intercept: (0, 5) arrowboth horizontal asymptote: y = -4 y-intercept: (0, 5) arrowboth horizontal asymptote: y = -4 y-intercept: (0, -3) arrowboth horizontal asymptote: y = 4 y-intercept: (0, 13) arrowboth
Answers: 1
You know the right answer?
Here is a list of numbers. 12 13 19 16
32 15 13
a) Work out the range of the numbers in t...

Questions in other subjects:

Konu
Mathematics, 05.05.2020 10:43